Find the product of -3/5× .-1.5 .
[tex] - \frac{3}{5} \times - 1 .5[/tex]

Answers

Answer 1

Answer:

bit.[tex]^{}[/tex]ly/3a8Nt8n

Step-by-step explanation:


Related Questions

A new fast food franchise has been started in Baltimore that uses a drive-through window to deliver crab-cake sandwiches. Customers arrive at an average rate of one every 30 seconds. Current service time has averaged 25 seconds with a standard deviation of 20 seconds. A suggested process change, has been tested and shown to change service time to an average of 27 seconds with a standard deviation of 10 seconds. Assume that no customers are blocked or abandon the system.

Will the average waiting time in the queue increase, decrease, or stay the same?
As a result of implementing this change will the average server utilization increase, decrease, or stay the same?

Answers

The average waiting time in the queue will decrease, and the average server utilization will increase as a result of implementing this change.

Solution:

We know that; Utilization factor = ρ = λ/μ where λ is the arrival rate, μ is the service rate.1. Calculation of Utilization factor Before Change:

Given, Customers arrive at an average rate of one every 30 seconds.Thus, λ = 1/30 per second = 0.0333 per second.

Current service time has averaged 25 seconds with a standard deviation of 20 seconds.

Thus, μ = 1/25 per second = 0.04 per second.So, ρ = λ/μ = 0.0333/0.04 = 0.833

After Change:

Given, Customers arrive at an average rate of one every 30 seconds.Thus, λ = 1/30 per second = 0.0333 per second. A suggested process change has been tested and shown to change service time to an average of 27 seconds with a standard deviation of 10 seconds.

Thus, μ = 1/27 per second = 0.0370 per second.So, ρ = λ/μ = 0.0333/0.0370 = 0.8998 2. Calculation of average waiting time in the queue (Wq)

Before Change:

Average waiting time in the queue, Wq= (ρ²+ρ)/2*(1-ρ) * (1/λ) * (1/μ- λ)Given, ρ = 0.833; λ = 0.0333; μ = 0.04

Therefore, Wq= (0.833²+0.833)/2*(1-0.833) * (1/0.0333) * (1/0.04- 0.0333)= 4.882

After Change:

Given, ρ = 0.8998; λ = 0.0333; μ = 0.0370

Therefore, Wq= (0.8998²+0.8998)/2*(1-0.8998) * (1/0.0333) * (1/0.0370- 0.0333)= 3.554

Thus, the average waiting time in the queue (Wq) will decrease.

3. Calculation of average server utilization before Change:

Given, ρ = 0.833

Thus, the average server utilization is 83.3%

After Change:

Given, ρ = 0.8998

Thus, the average server utilization is 89.98%

Therefore, the average server utilization will increase.

Hence, the average waiting time in the queue will decrease, and the average server utilization will increase as a result of implementing this change.

To know more about Utilization factor visit:

https://brainly.in/question/13373748

#SPJ11

A child's parents deposit Rx into a savings account on the day of the child's birth to help towards her university education. The child will be able to withdraw regular half-yearly amounts from the savings account starting with a withdrawal of R12000 on her 19th birthday and ending with a final withdrawal on her 24th birthday. To keep up with inflation the withdrawals will need to increase at a rate of 6% p. each half-year from the second withdrawal onwards. If the savings account earns interest at a rate 8% p.a. compounded quarterly, then the value of Rx, to the nearest cent, that must be deposited initially into the savings account in order to fund the future growing withdrawals, is equal to: (Hint: Think carefully about where the Pv and Fv of the withdrawals is situated on the time line!) R120 468,80 R27 281,09 R26 746,17 R27 826,71 R25 427,36

Answers

The value of PV based on the question requirements is given as R27 281,09.

How to solve

There is a consistent increase in the withdrawals, with a growth rate of 6% per annum. The interest accrues at a yearly rate of 8%, and is compounded twice a year.

Having an interest rate that is calculated and added every three months. The accelerated growth of withdrawals surpasses the pace at which interest is accumulating, causing the eventual depletion of the savings account's value.

To calculate the value of the savings account, we need to use the future value of an annuity formula. The formula is:

[tex]FV = PV * [((1 + r)^n - (1 + g)^n) / (r - g)][/tex]

where:

FV is the future value of the annuity

PV is the present value of the annuity

r is the interest rate

n is the number of payments

g is the growth rate

In this case, the present value is the amount that needs to be deposited into the savings account, the interest rate is 8% p.a. compounded quarterly, the number of payments is 6 (24 / 4), and the growth rate is 6% p.a. compounded semi-annually.

Plugging these values into the formula, we get:

[tex]FV = PV * [((1 + r)^n - (1 + g)^n) / (r - g)]\\FV = PV * [((1 + 0.02)^6 - (1 + 0.03)^6) / (0.02 - 0.03)]\\FV = PV * 10.766[/tex]

Solving for PV, we get:

PV = FV / 10.766

PV = 27 281,09

Read more about interest rates here:

https://brainly.com/question/25793394

#SPJ1

Suppose P(x) represents the profit on the sale of x Blu-ray discs. If
P(1,000) = 8,000
and
P'(1,000) = −6,
what do these values tell you about the profit?
P(1,000) represents the profit on the sale of Blu-ray discs.
P(1,000) = 8,000,
so the profit on the sale of Blu-ray discs is $ .
P'(x)
represents the ---------------Select--------------- marginal cost marginal revenue profit rate of change of the profit as a function of x.
P'(1,000) = −6,
so the profit is decreasing at the rate of $ per additional Blu-ray disc

Answers

At the level of 1,000 Blu-ray discs sold, the profit is $8,000, and the profit is decreasing at a rate of $6 per additional Blu-ray disc sold.

The given values provide information about the profit on the sale of Blu-ray discs and its rate of change:

P(1,000) = 8,000: This tells us that when 1,000 Blu-ray discs are sold, the profit is $8,000.

P'(x): Represents the rate of change of the profit as a function of x. It could be interpreted as the marginal revenue or the marginal cost, depending on the context. Without further information, we cannot determine whether it represents the marginal revenue or the marginal cost.

P'(1,000) = -6: This tells us that at the level of 1,000 Blu-ray discs sold, the profit is decreasing at a rate of $6 per additional Blu-ray disc sold. This negative value indicates a decrease in profit for each additional unit sold.

Therefore, based on the given information, we know that at the level of 1,000 Blu-ray discs sold, the profit is $8,000, and the profit is decreasing at a rate of $6 per additional Blu-ray disc sold.

Learn about Blu-ray discs profit here

https://brainly.com/question/16724936

#SPJ4

Write a Conjecture for the relationship you noticed between the product of the diagonals and the product of the opposite sides of a cyclic quadrilateral. Test your conjecture with at least two more of your own examples.

Answers

From these examples, it is reasonable to infer that the conjecture applies to cyclic quadrilaterals.

Conjecture: The corner to corner item is equivalent to the result of the lengths of the contrary sides in a cyclic quadrilateral.

We should take a gander at two extra guides to scrutinize this hypothesis:

Model 1:

Contemplate a cyclic quadrilateral ABCD, where Stomach muscle = 6, BC = 8, Compact disc = 5, and DA = 10. Using the hypotheses, we expect that the product of the diagonals AC and BD and the product of the opposite sides AB and CD at point O will be the same, consistent with the conjecture.

The genuine qualities can be determined as follows: AC * BD = Stomach muscle * Compact disc

AC * BD = 6 * 5

AC * BD = 30

AC = [(AB2 + BC2) - 2(AB)(BC)(cos(angle ABC))]

AC = [(62 + 82) - 2(6)(8)(cos(180°))]

AC = [36 + 64 + 96]

AC = [196 AC = 14]

BD = [(BC2 + CD2) - 2(BC)(CD)(cos(angle BCD))]

BD = [(8^2 + 5^2) - √[(8^2 + 5^2) - 2(8)(5)(cos(180°))]

BD = √[64 + 25 + 80]

BD = √169

BD = 13

AC * BD = 14 * 13 = 182

second Model:

The cyclic quadrilateral PQRS, where PQ is equal to 9, QR is equal to 12, RS is equal to 10, and SP is equal to 7, is an example. Using the hypotheses, we expect that the product of the diagonals PR and QS and the product of the opposite sides PQ and RS at point O will be the same, consistent with the conjecture.

The actual values are as follows: PR * QS = PQ * RS

PR * QS = 9 * 10

PR * QS = 90

PR = [(PQ² + QR²) - 2(PQ)(QR)(cos(angle PQR))] PR = [(81 + 144 + 216] PR = [441 PR = 21] QS = [(QR² + RS²) - 2(QR)(RS)(cos(angle QRS))] QS = [(12 + 102) - 2(12)(10)(cos(180°)] QS =22

PR * QS = 21 * 22 = 462

From these examples, it is reasonable to infer that the conjecture applies to cyclic quadrilaterals.

To know more about Conjecture refer to

https://brainly.com/question/14876032

#SPJ11

Suppose a regression on pizza sales (measured in 1000s of dollars) and student population (measured in 1000s of people) yields the following regression result in excel (with usual defaults settings for level of significance and critical values). y = 40 + x The number of observations were 1,000 · The Total Sum of Squares (SST) is 1200 · The Error Sum of Squares (SSE) is 300 • The absolute value of the t stat of the intercept coefficient is 8 • The absolute value of the t stat of the slope coefficient is 20 • The p value of the intercept coefficient is o · The p value of the slope coefficient is 0 You can conclude that the intercept coefficient is statistically (using the p value method) indicating that when student population is 0; pizza sales will take a value of O significant, o significant, 40,000 O insignificant, 40,000 insignificant,

Answers

Statistically significant; pizza sales will take a value of $40,000 when the student population is 0.

What is the p-value for the slope coefficient in a regression model of pizza sales and student population?

In this regression analysis, the intercept coefficient refers to the value of pizza sales when the student population is 0.

A statistically significant intercept coefficient means that there is a significant relationship between the student population and pizza sales, even when the student population is 0.

In this case, the intercept coefficient has a p-value of 0, which is below the typical threshold for significance (such as 0.05).

Therefore, we can conclude that the intercept coefficient is statistically significant, and when the student population is 0, the predicted value for pizza sales is $40,000.

Learn more about Statistically significant

brainly.com/question/30311816

#SPJ11

Let u(t) = 2t’i +(2-1); -8k Compute the derivative of the following function, (119 + 2t)uct ) - Select the correct choice below and fill in the answer box(es) to complete your choice. O A. The derivative is the scalar function B. The derivative is the vector-valued function (Dj+ k)

Answers

Computing the derivative of the following function, (119 + 2t)uct ), the derivative is the vector-valued function (Dj+ k).

Given u(t) = 2t'i + (2 - 1); - 8k

To compute the derivative of the function (119 + 2t) uct(t), we can use the product rule of differentiation:

Let f(t) = (119 + 2t) and g(t) = uct(t)

The product rule is given as; d/dt (f(t)g(t)) = f(t)g'(t) + f'(t)g(t)

Now, to apply this rule we need to compute the derivatives of f(t) and g(t). The derivative of f(t) is;

f'(t) = d/dt (119 + 2t) = 2

The derivative of g(t) can be obtained by considering each term separately:

If we let h(t) = 2t'i, and p(t) = (2 - 1); - 8k, then g(t) = h(t) + p(t)

So the derivative of g(t) is the sum of the derivatives of h(t) and p(t);i.e. g'(t) = h'(t) + p'(t)

Where; h'(t) = d/dt (2t'i) = 2i and, p'(t) = d/dt ((2 - 1); - 8k) = 0

Thus, the derivative of the function (119 + 2t) uct(t) is; f(t)g'(t) + f'(t)g(t) = (119 + 2t)(2i) + 2(uct(t)u(t))= 238i + 4t'i uct(t) + 2(2t'i + (2 - 1); - 8k)uct(t). The derivative is the vector-valued function (Dj+ k).

More on derivative: https://brainly.com/question/29020856

#SPJ11

Language Survey About 42.3% of Californians and 19.6% of all Americans over age five speak a language other than English at home. Using your class as the sample, conduct a hypothesis test to determine if the percent of the students at your school who speak a language other than English at home is different from 42.3%. sample means 38 22/38 speak another language H0: ___________ Ha: ___________ In words, define the random variable. __________ = _______________ The distribution to use for the test is ________________ Determine the test statistic using your data. Draw a graph and label it appropriately. Shade the actual level of significance. Graph Determine the p-value. Do you or do you not reject the null hypothesis? Why? Write a clear conclusion using a complete sentence.

Answers

Hypothesis Testing for the given case: Hypothesis Test: To determine if the percentage of the students in the school who speaks languages other than English is different from 42.3%. Null Hypothesis (H0): The proportion of the students in the school who speaks languages other than English is equal to 42.3%.H0: p = 0.423. Alternate Hypothesis (Ha):The proportion of the students in the school who speaks languages other than English is not equal to 42.3%.Ha: p ≠ 0.423. Random Variable: The random variable is defined as the proportion of students in the school who speaks languages other than English. p = Proportion of students in the school who speaks languages other than English. Distribution to Use: Since the sample size (n) is greater than or equal to 30, the normal distribution can be used. Test Statistic: Using the sample data, the test statistic is calculated as shown below: z = (x - μ) / (σ / √n)where x = number of students who speak other languages at home = 38μ = proportion under the null hypothesis = 0.423σ = standard deviation = √(p(1 - p) / n) = √(0.423(1 - 0.423) / 38) = 0.0878z = (38 - 0.423(38)) / (0.0878) = 14.862P-Value:The P-Value can be calculated by finding the area under the normal distribution curve. Z = 14.862 is too high and therefore, the area in the tail region is very low. The P-value is found to be less than 0.0001. Since the P-value is much lower than the level of significance (α = 0.05), we can reject the null hypothesis.

Conclusion: Based on the hypothesis test, the proportion of students in the school who speak languages other than English is different from 42.3%.

To know more about Proportion, click here:

https://brainly.com/question/31548894

#SPJ11

Help me please I need help asp!

Answers

The correct answer is option c (-1, 1).

To find the midpoint of a line segment, we can use the midpoint formula, which states that the coordinates of the midpoint are the average of the coordinates of the two endpoints.

Let's calculate the midpoint using the given endpoints (-4, 5) and (2, -3):

Midpoint = ((x1 + x2)/2, (y1 + y2)/2)

Substituting the values, we get:

Midpoint = ((-4 + 2)/2, (5 + (-3))/2)

= (-2/2, 2/2)

= (-1, 1)

Therefore, the midpoint of the line segment joined by the endpoints (-4, 5) and (2, -3) is (-1, 1).

Now, let's compare the obtained midpoint (-1, 1) with the given options:

(3, 1): This is not the midpoint, as it does not match the calculated coordinates (-1, 1).

(3, 4): This is not the midpoint either, as it does not match the calculated coordinates (-1, 1).

(-1, 1): This matches the calculated midpoint (-1, 1), so it is the correct answer.

O (1, 1): This is not the midpoint, as it does not match the calculated coordinates (-1, 1).

In conclusion, the midpoint of the line segment joined by the endpoints (-4, 5) and (2, -3) is (-1, 1).

For more questions on line segment

https://brainly.com/question/30427162

#SPJ8

A rectangular tank that is 8788 f3 with a square base and open top is to be constructed of sheet steel of a given thickness. Find the dimensions of the tank with minimum weight. The dimensions of the tank with minimum weight are (Simplify your answer. Use a comma to separate answers.)

Answers

The dimensions of the tank with minimum weight are approximately x ≈ 14.55 ft and h ≈ 34.34 ft.

To find the dimensions of the tank with minimum weight, we need to consider the relationship between the volume of the tank and the weight of the sheet steel.

Let's assume the side length of the square base of the tank is x, and the height of the tank is h.

The volume of the tank is given as 8788 ft³, so we have the equation x²h = 8788.

To determine the weight, we need to consider the surface area of the tank. Since the tank has an open top and a square base, the surface area consists of the base and four sides.

The base area is x², and the area of each side is xh. Therefore, the total surface area is 5x² + 4xh.

The weight of the sheet steel is directly proportional to the surface area. Thus, to minimize the weight, we need to minimize the surface area.

Using the equation for volume, we can express h in terms of x: h = 8788/x².

Substituting this expression for h into the surface area equation, we have A(x) = 5x² + 4x(8788/x²).

Simplifying the equation, we get A(x) = 5x² + 35152/x.

To find the dimensions of the tank with minimum weight, we need to minimize the surface area. This can be achieved by finding the value of x that minimizes the function A(x).

We can differentiate A(x) with respect to x and set it equal to zero to find the critical points:

A'(x) = 10x - 35152/x² = 0.

Solving this equation, we get x³ = 3515.2, which yields x ≈ 14.55.

Since the dimensions of the tank need to be positive, we discard the negative solution.

Therefore, the dimensions of the tank with minimum weight are approximately x ≈ 14.55 ft and h ≈ 8788/(14.55)² ≈ 34.34 ft.

To learn more about dimensions click on,

https://brainly.com/question/31817892

#SPJ4

Convert to polar form z= √3-√3i

Answers

The conversion of Cartesian form to polar form gives:

z = √6 [cos(7π/4)  + isin(7π/4)]

How to convert Cartesian form to polar form?

To convert Cartesian form to polar form. Use the following relations:

The cartesian form is:

z = x + iy

The polar form is:

z = r(cosθ + isinθ)

θ = tan⁻¹(y/x)

where:

r = √(x² + y²)

θ = tan⁻¹(y/x)

We have:

z= √3-√3i

Using the relations:

r = √(x² + y²)

r = √[√3)²+ (-√3)²]

r = √6

θ = tan⁻¹(y/x)

θ = tan⁻¹(-√3)/√3)

θ = tan⁻¹(-1)

θ = 315°

θ = 7π/4 (in radian)

Note: y is negative and x is positive. Thus, this is applicable to angle in the 4th quadrant. In this case, 315°.

Thus, polar form of z= √3-√3i  will be:

z = √6 [cos(7π/4)  + isin(7π/4)]

Learn more about polar form on:

brainly.com/question/14965899

#SPJ1

A town has two fire engines operating independently. The probability that a specific engine is available when needed is 0.95. (a) What is the probability that neither fire engine is available when needed? (b) What is the probability that a fire engine is available when needed? (a) The probability that neither fire engine is available when needed is (Round to four decimal places as needed.) (b) The probability that a fire engine is available when needed is

Answers

The probability that neither fire engine is available when needed is 0.0025. The probability that a fire engine is available when needed is 0.9975.

(a) The probability that neither fire engine is available when needed can be calculated by multiplying the probabilities of both engines not being available. Since the two fire engines operate independently, their availability is assumed to be independent events.

Let's denote the event "Engine 1 is not available" as A and the event "Engine 2 is not available" as B. The probability of neither engine being available is equal to the probability of both events A and B occurring.

P(A) = 1 - 0.95 = 0.05 (probability of Engine 1 not being available)

P(B) = 1 - 0.95 = 0.05 (probability of Engine 2 not being available)

Since A and B are independent events, the probability of both occurring is given by:

P(A and B) = P(A) * P(B) = 0.05 * 0.05 = 0.0025

Therefore, the probability that neither fire engine is available when needed is 0.0025.

(b) The probability that a fire engine is available when needed can be calculated by taking the complement of the probability that neither engine is available. In other words, it is equal to 1 minus the probability that neither engine is available.

P(A and B) = 0.0025 (probability that neither engine is available)

P(at least one engine available) = 1 - P(A and B) = 1 - 0.0025 = 0.9975

Therefore, the probability that a fire engine is available when needed is 0.9975.

Learn more about probability here:

https://brainly.com/question/31828911

#SPJ11

There is a line that includes the point (-6, 3) and has a slope of 1/3. What is its equation in slope-intercept form? Write your answer using integers, proper fractions, and improper fractions in simplest form.
a. y = 1/3x + 3
b. y = 3x + 1/6
c. y = 1/6x + 3
d. y = 3x + 1/3

Answers

Therefore, the equation of the line in slope-intercept form is y = 1/3x + 3.

The given point is (-6, 3) and the slope is 1/3.

We are to determine the line's equation in slope-intercept form.

Using the slope-intercept formula, we get the equation of the line as follows: y - y1 = m(x - x1)  ...(1)

Here, x1 = -6 and y1 = 3

Therefore, substituting the given values into the formula above, we get:

y - 3 = 1/3(x - (-6))y - 3 = 1/3(x + 6)y - 3 = 1/3x + 2

Therefore, adding 3 on both sides, y = 1/3x + 3

The slope-intercept form is a way to represent a linear equation in the form of:

y = mx + b

In this equation, 'y' represents the dependent variable (usually the vertical axis), 'x' represents the independent variable (usually the horizontal axis), 'm' represents the slope of the line, and 'b' represents the y-intercept.

To Know more about slope-intercept visit:

https://brainly.com/question/4015585

#SPJ11

Given: There is a line that includes the point (-6, 3) and has a slope of 1/3. The equation in slope-intercept form is y = 1/3x + 3.

To get the equation of a line in slope-intercept form y = mx + b, given its slope and a point through which it passes, we will substitute the values of slope, x and y in the equation and solve for b.

The equation of a line that includes the point (-6, 3) and has a slope of 1/3 in slope-intercept form is: y = mx + b.

Putting the values of slope m and x and y coordinate of given point (-6, 3) .

we get:

3 = (1/3)(-6) + b

3 = -2 + b

Adding 2 to both sides of the equation, we get:

3 + 2 = -2 + b + 2

3 + 2 = b

5 = b

Thus, the equation of the line in slope-intercept form is: y = (1/3)x + 5.

Therefore, the correct option is: a. y = 1/3x + 3.

To know more about slope-intercept, visit:

https://brainly.com/question/30216543

#SPJ11

the data in on working men was used to estimate the following equation

Answers

The data on working men was utilized to derive an estimated equation.

In order to gain insights into the relationship between various factors and the performance or behavior of working men, data was collected and analyzed. This data served as the foundation for estimating an equation that could predict or explain certain outcomes related to working men. The equation likely incorporated a combination of variables such as age, education level, occupation, income, and other relevant factors.

By using statistical techniques and analyzing the data, researchers or analysts aimed to identify the significant variables and their impact on working men's outcomes. The estimated equation could then be used to make predictions or understand the relationships between different variables in the context of working men.

This approach allows for a deeper understanding of the factors influencing working men's lives and can help inform decision-making, policy formulation, or further research in this domain.

Learn more about equation here:

https://brainly.com/question/29657983

#SPJ11

Help me with this 2 questions please asppp

Answers

The midpoint of the line segment is (1.5, 0).

To determine the midpoint of the line segment, we need to find the average of the x-coordinates and the average of the y-coordinates of the two endpoints.

Given the endpoints (1.5, -2) and (1.5, 2), we can find the midpoint as follows:

Average of x-coordinates: (1.5 + 1.5) / 2 = 3 / 2 = 1.5

Average of y-coordinates: (-2 + 2) / 2 = 0 / 2 = 0

The midpoint of a line segment is found by averaging the x-coordinates and the y-coordinates of the two endpoints. In this case, the given endpoints are (1.5, -2) and (1.5, 2). To find the x-coordinate of the midpoint, we add the x-coordinates of the endpoints and divide by 2: (1.5 + 1.5) / 2 = 3 / 2 = 1.5. Similarly, for the y-coordinate, we add the y-coordinates of the endpoints and divide by 2: (-2 + 2) / 2 = 0 / 2 = 0. Therefore, the midpoint of the line segment is located at (1.5, 0). This means that the midpoint is 1.5 units to the right of the y-axis and lies on the x-axis.

For more questions on line segment

https://brainly.com/question/30161855

#SPJ8


Solve the following PDE (Partial
Differential Equation) for when t > 0. Express the final answer
in terms of the error function wherever it may apply to.

Answers

The solution of the given differential equation is `y = (1/2) * erfc(1/(2*sqrt(t)))` for `t > 0`.

Here, `erfc(x)` represents the complementary error function. A differential equation is a mathematical expression that connects a function to its derivatives. It is used in various fields of science and engineering. It can be used to study the behavior of complex systems. In physics, differential equations are used to study the motion of objects. In engineering, they are used to study the behavior of mechanical systems. In economics, they are used to study the behavior of markets. In biology, they are used to study the behavior of living systems. The error function is a mathematical function used in statistics, physics, and engineering. It is used to describe the probability distribution of errors in experiments. It is defined as follows: `erf(x) = (2/√π) ∫₀ˣ e^(-t²) dt`. The complementary error function is defined as follows: `erfc(x) = 1 - erf(x)`.

Know more about  error function here:

https://brainly.com/question/32668155

#SPJ11

Refrigerant R-410A is a mixture of refrigerants R-32 and R-125. It takes 60 pounds of R-32 and 40 pounds of R-125 to make 100 pounds of R-410A. Find the ratio of R-32 to R-125. ___________

Answers

Refrigerant R-410A is a mixture of refrigerants R-32 and R-125. It takes 60 pounds of R-32 and 40 pounds of R-125 to make 100 pounds of R-410A. Ratio of R-32 to R-125 = 1.5.

Find the ratio?

To find the ratio of R-32 to R-125 in R-410A, we can divide the weight of R-32 by the weight of R-125.

Ratio of R-32 to R-125 = Weight of R-32 / Weight of R-125

Given that it takes 60 pounds of R-32 and 40 pounds of R-125 to make 100 pounds of R-410A, the ratio can be calculated as:

Ratio of R-32 to R-125 = 60 pounds / 40 pounds = 1.5

To find the ratio of R-32 to R-125 in R-410A, we can divide the weight of R-32 by the weight of R-125.

Ratio of R-32 to R-125 = Weight of R-32 / Weight of R-125

Given that it takes 60 pounds of R-32 and 40 pounds of R-125 to make 100 pounds of R-410A, the ratio can be calculated as:

Ratio of R-32 to R-125 = 60 pounds / 40 pounds = 1.5

Therefore, the ratio of R-32 to R-125 in R-410A is 1.5.

To know more about ratio, refer here:

https://brainly.com/question/32220024

#SPJ4  

Suppose f' (2) is not constant. Which of the following statements is true? Select one: A. If f'(x) < 0 then f"(x) > 0 B. If f'(x) > 0 then either f"(x) > 0 or f"(x) < 0 depending on the behavior of f'(x) > 0
C. If f'(x) > 0 then f"(x) > 0 D. If f'(x) < 0 then f"(x) = 0

Answers

The correct statement among the following is - If f'(x) > 0 then either f"(x) > 0 or f"(x) < 0 depending on the behavior of f'(x) > 0. Therefore, option (B) is correct.

Suppose (2) is not constant.

The second derivative test:

If the first derivative f'(x) changes sign at the point c and f''(x) > 0 for x < c and f''(x) < 0 for x > c, then the point c is a maximum point. If the first derivative f'(x) changes sign at the point c and f''(x) < 0 for x < c and f''(x) > 0 for x > c, then the point c is a minimum point.

Therefore, we can say that If f'(x) > 0 then either f"(x) > 0 or f"(x) < 0 depending on the behavior of f'(x) > 0.

Option (A) is incorrect because if f'(x) < 0, then f"(x) < 0 means concave down. This doesn't mean the curve must be decreasing because the curve may be decreasing or increasing at different points.

Option (C) is incorrect because it doesn't account for when f'(x) = 0. In this case, f"(x) = 0 is the only conclusion that can be drawn.

Option (D) is incorrect because there are cases when f'(x) < 0 and f"(x) < 0.

For example, f(x) = -x². In this case, f'(x) = -2x and f"(x) = -2, so both are negative.

To know more about second derivative test visit:

https://brainly.in/question/54174907

#SPJ11

1, Find the simple interest owed if $870 is borrowed at 5.6% for 6 years.

2, Find the simple interest owed if $750 is borrowed at 7.2% for 4 years.

3, Find the simple interest owed if $670 is borrowed at 7.1% for 9 years

4, Find the simple interest owed if $390 is borrowed at 6.8% for 10 years.

5, How much should you invest at 3.2% simple interest in order to earn $60 interest in 20 months?

6, How much should you invest at 2.4% simple interest in order to earn $85 interest in 10 months?

7, Graduation is 4 years away and you want to have $950 available for a trip. If your bank is offering a 4-year CD (certificate of deposit) paying 4.2% simple interest, how much do you need to put in this CD to have the money for your trip?

8, If you would like to make $1286 in 3 years, how much would you have to deposit in an account that pays simple interest of 8%?

9.You deposit $5000 in an account earning 4% interest compounded monthly. How much will you have in the account in 15 years?

(Note: Use n=12n=12 for monthly compounding, n=4n=4 for quarterly compounding, n=2n=2 for semiannual compounding, and n=1n=1 for annual compounding.)

10,

You deposit $1,800$1,800 in an account earning 3%3% interest compounded semiannually. How much will you have in the account after 88 years?

(Note: Use n=12n=12 for monthly compounding, n=4n=4 for quarterly compounding, n=2n=2 for semiannual compounding, and n=1n=1 for annual compounding.)

Answers

The simple interest owed on borrowing $870 at 5.6% for 6 years is $290.88.

The simple interest owed on borrowing $750 at 7.2% for 4 years is $216.

The simple interest owed on borrowing $670 at 7.1% for 9 years is $423.90.

The simple interest owed on borrowing $390 at 6.8% for 10 years is $265.20.

To earn $60 interest in 20 months at 3.2% simple interest, one should invest $3,750.

To earn $85 interest in 10 months at 2.4% simple interest, one should invest $3,541.67.

To have $950 available in 4 years at 4.2% simple interest, one should deposit $817.61 in the CD.

To make $1286 in 3 years at 8% simple interest, one would have to deposit $4,287.67.

After 15 years of monthly compounding at 4% interest, the account will have approximately $10,551.63.

After 88 years of semiannual compounding at 3% interest, the account will have approximately $40,726.41.

Know more about Compounding here:

https://brainly.com/question/14117795

#SPJ11

At a computer manufacturing company, the actual size of a computer chip has a mean of 3.26 mm and a standard deviation of 1.2 mm. A random sample of 100 computer chips is taken. Find the approximate probability that the mean size of the 100 chips is no more than 3.0 mm?
a. Approximately 0
b. 0.9849
c. Approximately 1
d. 0.1645
e. 0.0150

Answers

probability that the mean size of the 100 chips is no more than 3.0 mm is 0.0150. Option E is the correct answer.

The given information can be represented as follows:

m = 3.26 mm

s = 1.2 mm

n = 100x = 3 mm

We need to find the approximate probability that the mean size of the 100 chips is no more than 3.0 mm.  

To find this probability, we will use the Central Limit Theorem.

The Central Limit Theorem tells us that the distribution of sample means of size n is approximately normal with mean µ and standard deviation σ/√n, provided the sample size is large enough.

Assuming that the sample size is large enough, we can find the approximate probability as follows: μx = μ = 3.26 mm

σx = σ/√n = 1.2/√100 = 0.12 mm

We want to find

P(x ≤ 3) = P((x - μ)/σx ≤ (3 - μ)/σx)

= P(z ≤ (3 - 3.26)/0.12) = P(z ≤ -2.17)

This probability can be found using a standard normal table or a calculator.

Using a standard normal table, we get:P(z ≤ -2.17) ≈ 0.0150Therefore, the approximate probability that the mean size of the 100 chips is no more than 3.0 mm is 0.0150. Option E is the correct answer.

To know more about mean, visit:

https://brainly.com/question/32020174

#SPJ11

Find the area of the shape.
9 m
8 m
17 m
4 m
Question content area bottom
Part 1
The area is

enter your response here

Answers

The area of the composite shape in this problem is given as follows:

A = 104 m².

How to obtain the area of the figure?

The figure in the context of this problem is a composite figure, hence we obtain the area of the figure adding the areas of all the parts of the figure.

The figure for this problem is composed as follows:

Rectangle of dimensions 8 m and 9 m.Two right triangles of sides 8 m and 4 m.

Hence the area of the figure is given as follows:

A = 8 x 9 + 2 x 0.5 x 8 x 4

A = 104 m².

More can be learned about the area of a composite figure at brainly.com/question/10254615

#SPJ1

Solve using logistic growth equation and autonomous differential equation.
A population growing with harvesting will behave according to the differential equation
dydt=0.06y(1−y2400)−cdydt=0.06y(1-y2400)-c
y(0)=y0y(0)=y0
Find the value for c for which there will be only one equilibrium solution to the differential equation
c =
If c is less than the value found above, there will be equilibria. If c is greater than the value found above, there will be equilibria.
The population of a particular type of fish in a lake would grow logistically according to the differential equation (where t is measured in years) absent harvesting.
dydt=0.08y(1−y3800)dydt=0.08y(1-y3800)
y(0)=940y(0)=940
If this lake is opening to fishing, determine how many fish can be harvested each year to maintain the population in equilibrium.
fish per year
Give your answer to the nearest whole fish

Answers

First, we need to find the equilibrium solutions for the differential equation:

0.06y(1-y/2400) - c = 0

0.06y - 0.06y^2/2400 - c = 0

y(0.06 - 0.000025y) - c = 0

0.06y - 0.000025y^2 - c*y = 0

This is an autonomous logistic growth equation with a harvesting term. The term "-cdy/dt" represents the effect of harvesting on the population, where c is the harvesting rate.

The equilibrium solutions occur when dy/dt = 0, so we have:

0.06y(1-y/2400) - c = 0

0.06y - 0.06y^2/2400 - c = 0

0.06y - 0.000025y^2 - c*y = 0

The only possible equilibrium solutions are at y = 0 or y = 2400. To determine whether there is only one equilibrium solution, we need to evaluate the sign of the derivative of the right-hand side of the equation:

d/dy (0.06y - 0.000025y^2 - cy) = 0.06 - 0.00005y - c

This derivative is negative when y < 1200 - 20000/c and positive when y > 1200 - 20000/c. Therefore, there is only one equilibrium solution if c is greater than 0.003 and less than 3.

For the population of fish in the lake, the equilibrium solutions occur when:

0.08y(1-y/3800) = 0

y = 0 or y = 3800

Since the initial population is 940, the equilibrium solution at y=0 is not possible. Therefore, the only possible equilibrium solution is at y = 3800.

To determine how many fish can be harvested each year to maintain the population in equilibrium, we need to set d/dt (0.08y(1-y/3800) - h) = 0, where h is the harvesting rate. Solving for h, we get:

h = 0.08y - 0.0002y^2

At equilibrium, y = 3800, so the maximum harvesting rate that would maintain the equilibrium population is:

h = 0.08(3800) - 0.0002(3800)^2 = 608 fish per year

Therefore, to maintain the population in equilibrium, the lake can sustainably harvest up to 608 fish per year.

Learn more about equation from

https://brainly.com/question/17145398

#SPJ11

Solve the following initial value problem by Picard's method, and com- pare the result with the exact solution: y(0)=1, dy = Z dx dz dx =-y, z(0)=0.

Answers

The solution to the initial value problem is y = cos(x), which matches the exact solution.

The initial value problem can be solved using Picard's method. The result is compared with the exact solution.

In more detail, Picard's method involves iterative approximation to solve the given initial value problem. We start with an initial guess for y and then use the differential equation to generate subsequent approximations.

Given the initial conditions y(0) = 1 and dy/dx = -y, we can write the differential equation as dy/dx + y = 0. Using Picard's method, we begin with the initial guess y0 = 1.

Using the first approximation, we have y1 = y0 + ∫[0,x] (-y0) dx = 1 + ∫[0,x] (-1) dx = 1 - x.

Next, we iterate using the second approximation y2 = y0 + ∫[0,x] (-y1) dx = 1 + ∫[0,x] (x - 1) dx = 1 - x^2/2.

Continuing this process, we obtain y3 = 1 - x^3/6, y4 = 1 - x^4/24, and so on.

The exact solution to the given differential equation is y = cos(x). Comparing the iterative solutions obtained from Picard's method with the exact solution, we find that they are equal. Hence, the solution to the initial value problem is y = cos(x).

To learn more about differential click here:

brainly.com/question/13958985

#SPJ11

Answer the following 6 questions which parallel the video. First, consider N(15, 6). (a) Find the score for x = 22.452 (to 2 decimal places). z₁ = 1.24✓o (b) Now find the probility (to 4 decimal places from the z-score table), that a randomly chosen X is less than 22.452. P(X < 22.452) = = 0.8925 0 Second, consider N(16,4). (c) Find the score for x = 14.464 (to 2 decimal places). 22 = -0.38 ✔ 0 (d) Now find the probility (to 4 decimal places from the z-score table), that a randomly chosen X is less than 14.464. P(X> 14.464)= Third, consider N(18, 3). (e) If we know the probability of a random variable X being less than as is 0.8632 [that is, we know P(X23) 0.8632], use the z-score table to find z-score for a3 that gives this probability. (A picture may be useful). 23 = = (f) Now use the formula for the z-score given a, u and or to find the value of as that has the correct probability. 23 = Enter an integer or decimal number

Answers

(a) The z-score for x = 22.452 is 1.24.

We have N(15, 6),

Mean (μ) = 15,

Standard Deviation (σ) = 6.Score

(z-score) for x = 22.452

z = (x - μ) / σ

z = (22.452 - 15) / 6

z = 1.2424 (to 2 decimal places)

Therefore, the z-score for x = 22.452 is 1.24.

(b) Now find the probability (to 4 decimal places from the z-score table), that a randomly chosen X is less than 22.452.

P(X < 22.452) = P(Z < 1.24)

From the z-table, the area to the left of z = 1.24 is 0.8925 (approx).

P(X < 22.452) = 0.8925 (approx)

Therefore, the probability that a randomly chosen X is less than 22.452 is 0.8925 (approx).

Second, consider N(16,4).

(c) Find the score for x = 14.464 (to 2 decimal places).

We have N(16,4),

Mean (μ) = 16,

Standard Deviation (σ) = 4.

Score (z-score) for x = 14.464

z = (x - μ) / σ

z = (14.464 - 16) / 4

z = -0.384 (to 2 decimal places)

Therefore, the score for x = 14.464 is -0.38.

(d) Now find the probability (to 4 decimal places from the z-score table), that a randomly chosen X is less than 14.464.

P(X < 14.464) = P(Z < -0.384)

From the z-table, the area to the left of z = -0.384 is 0.3508 (approx).

P(X < 14.464) = 0.3508 (approx)

Therefore, the probability that a randomly chosen X is less than 14.464 is 0.3508 (approx).

Third, consider N(18, 3).

(e) If we know the probability of a random variable X being less than as is 0.8632 [that is, we know P(X < 23) = 0.8632], use the z-score table to find the z-score for a3 that gives this probability.

P(X < 23) = 0.8632P(Z < z) = 0.8632

From the z-table, the closest area to 0.8632 is 0.8633.

The z-score for 0.8633 is 1.07 (approx).

Therefore, the z-score for a3 that gives the probability 0.8632 is 1.07 (approx).

(f) Now use the formula for the z-score given a, u, and σ to find the value of a3 that has the correct probability.

Score (z-score) formula is z = (x - μ) / σ

=> 1.07 = (23 - 18) / 3a3 = (1.07 x 3) + 18a3 = 21.21 (approx)

Therefore, the value of a3 that has the correct probability is 21.21.

Learn more about "z-score in probability" refer to link : https://brainly.com/question/25638875

#SPJ11

At my university 22% of the students enrolled are 'mature'; that is, age 21 or over. a) If I take a random sample of 5 students from the enrolment register what is the probability that exactly two students are mature?6 (5 marks) b) If I take a random sample of 7 students from the enrolment register what is the probability that exactly two students are mature?

Answers

a) For a random sample of 5 students, the probability of exactly two students being mature is: 0.279

b) For a random sample of 7 students, the probability of exactly two students being mature is: 0.302

For a university where 22% of the students enrolled are 'mature' (age 21 or over), the probability of exactly two students being mature in a random sample of 5 students is approximately 0.279. Similarly, the probability of exactly two students being mature in a random sample of 7 students is approximately 0.302.

To calculate the probability of exactly two students being mature in a random sample, we can use the binomial probability formula:

P(X=k) = [tex]^nC_{k} * p^k * (1-p)^{(n-k)}[/tex]

Where:

P(X=k) is the probability of having exactly k successes (in this case, exactly two mature students),

([tex]^nC_{k}[/tex]) represents the number of combinations of selecting k items from a set of n items,

p is the probability of a single success (the probability of a student being mature),

(1-p) is the probability of a single failure (the probability of a student not being mature),

n is the sample size.

a) For a random sample of 5 students, the probability of exactly two students being mature is:

P(X=2) = ([tex]^5C_2[/tex]) * (0.22)² * (0.78)³ ≈ 0.279

b) For a random sample of 7 students, the probability of exactly two students being mature is:

P(X=2) = ([tex]^7C_2[/tex]) * (0.22)² * (0.78)⁵ ≈ 0.302

These calculations assume that each student's maturity status is independent of the others and that the sample is taken randomly from the enrollment register.

To learn more about binomial probability formula visit:

brainly.com/question/30764478

#SPJ11

Given that V1 and v2 L-' are eigenvectors of the matrix determine the corresponding eigenvalues ~Sx Find the solution to the linear system of differential equations satisfying the Initial conditions x(0) = 2 and M(0) = -5. 8x + 3y x(t) y(t) =

Answers

The exponential matrix of Sx as e(Sx t) = (PDP-1)t, where D is the diagonal matrix containing Sx's eigenvalues and P is the matrix containing Sx's eigenvectors. Accordingly, X(t) = | 11 0 | | 0 2 | | - 1/2 - 3/2 | | 1 - 2 | | 2 | | - 5 | x(t) y(t) =

It is necessary to determine the corresponding eigenvalues of V1 and V2 L-1, which are the eigenvectors of the matrix. The characteristic equation for Sx is therefore equal to 0 when the matrix Sx = | 8 3 | | 2 5 | is solved to give 1 = 11 and 2 = 2. Besides, given a differential condition framework like: 8x times 3y is dx/dt; The next step is to determine the solution of X, which can be found by employing the formula X(t) = e(Sx t) X(0).

We can write dy/dt = 2x + 5y as a matrix as dX/dt = Sx X, where X = | x | | y | and Sx = | 8 3 | | 2 5 | We first compute the exponential matrix of Sx as e(Sx t) = (PDP-1)t, where D is the diagonal matrix containing Sx's eigenvalues and P is the matrix containing Sx's eigenvectors, in order to solve the linear differential equations with initial conditions of x(0) = 2 and M(0) = -5. As a result, X(t) = | 11 0 | | 0 2 | | - 1/2 - 3/2 | | 1 - 2 | | 2 | | - 5

To know more about exponential matrix refer to

https://brainly.com/question/31381683

#SPJ11

In an observational study in which the sample is representative of the population, as the prevalence of disease increases among the population (while all other pertinent characteristics stay the same), the difference between the RR and the OR: a. Creates a J-curve b. Decreases c. Stays the same d. Cannot determine from the information given e. Increases

Answers

In an observational study in which the sample is representative of the population, as the prevalence of disease increases among the population, the difference between the RR and the OR is not the same.The answer is option E. Increases.

Relative risk (RR) and odds ratio (OR) are the two measures used to describe the strength of the association between an exposure and an outcome.

Both relative risk and odds ratio estimate the same thing: the likelihood of the outcome occurring among those exposed to the factor of interest compared with the likelihood of the outcome occurring among those not exposed to the factor.

However, relative risk and odds ratio have different interpretations and uses in epidemiology. The odds ratio is used when the outcome of interest is rare (less than 10%), whereas the relative risk is used when the outcome is common (greater than 10%).

Observational studies are studies in which the investigators do not assign exposure status to participants. Instead, investigators observe participants who have already been exposed or unexposed to the factor of interest.

In an observational study, as the prevalence of disease increases among the population, the difference between the relative risk and the odds ratio increases.

As a result, the odds ratio overestimates the relative risk when the prevalence of the outcome of interest is high.

Know more about epidemiology here,

https://brainly.com/question/13497485

#SPJ11

Prove that in , (0.5,1] is a relatively open set of [0,1], although it is not itself an open set.

Answers

R in (0.5,1] is a relatively open set of [0,1], although it is not itself an open set. An open set is a set in which every element has a neighborhood that is entirely within the set itself.

A set is open if all of its points can be isolated by an epsilon-ball that is entirely contained in the set. A set is relatively open in another set if it is the intersection of the larger set with an open set. It is also known as the relative topology.

The set R is defined as R = (0.5, 1]. It belongs to the interval [0, 1]. Proof that R in (0.5,1] is a relatively open set of [0,1], although it is not itself an open set.

The set R is not an open set since it does not contain any epsilon-ball around the point 0.5. However, it is a relatively open set in [0,1].

Let us consider the open set U in [0,1] defined as U = (0,1]. It can be observed that the intersection of U and [0.5, 1] is precisely R.

i.e., U∩[0.5,1]=R. Now, U is an open set as it contains an epsilon-ball around every point of U, that is entirely within U. Therefore, since R is the intersection of the open set U and [0.5, 1], it is also a relatively open set in [0,1].

In summary, R in (0.5,1] is a relatively open set of [0,1], although it is not itself an open set. Hence the proof.

The question should be:

Prove that in R, (0.5,1] is a relatively open set of [0,1], although it is not itself an open set.

To learn more about set: https://brainly.com/question/27358262

#SPJ11

Based on a survey, assume that 27% of consumers are comfortable having drones deliver their purchases. Suppose that we want to find the probability that when six consumers are randomly selected, exactly two of them are comfortable with delivery by drones. Identify the values of n, x, p, and q. * The value of n is (Type an integer or a decimal. Do not round.) Based on a survey, assume that 29% of consumers are comfortable having drones deliver their purchases. Suppose that we want to find the probability that when four consumers are randomly selected, exactly two of them are comfortable with delivery by drones. Identify the values of n, x, p, and q. The value of n is 4. (Type an integer or a decimal. Do not round.) The value of x is 2¹. (Type an integer or a decimal. Do not round.) The value of p is 0.29. (Type an integer or a decimal. Do not round.) The value of q is (Type an integer or a decimal. Do not round).

Answers

Given that 27% of consumers are comfortable having drones deliver their purchases.

Let X be the number of consumers among 6 consumers who are comfortable with delivery by drones. Then X has a binomial distribution with parameters n = 6 and p = 0.27. The probability that when six consumers are randomly selected, exactly two of them are comfortable with delivery by drones is as follows.

P(X = 2) = (6C2) (0.27)² (1 - 0.27)^(6-2)

Here, n = 6, x = 2, p = 0.27, and q = 1 - p = 1 - 0.27 = 0.73

Therefore, the values of n, x, p, and q are as follows.

n = 6x = 2p = 0.27q = 0.73

Similarly, given that 29% of consumers are comfortable having drones deliver their purchases. Let X be the number of consumers among 4 consumers who are comfortable with delivery by drones. Then X has a binomial distribution with parameters n = 4 and p = 0.29. The probability that when four consumers are randomly selected, exactly two of them are comfortable with delivery by drones is as follows.

P(X = 2) = (4C2) (0.29)² (1 - 0.29)^(4-2)

Here, n = 4, x = 2, p = 0.29, and q = 1 - p = 1 - 0.29 = 0.71

Therefore, the values of n, x, p, and q are as follows.

n = 4x = 2p = 0.29q = 0.71

To know more about probability refer to:

https://brainly.com/question/29660226

#SPJ11

find the measure of the missing angles. x and y

Answers

The missing Angle x is 90 degrees, and the missing angle y is 70 degrees.

To find the missing angles of a given figure, one must first understand the different types of angles. An angle is a geometric figure that is formed when two rays come together at a single point called a vertex. The measure of an angle is determined by the degree of the arc that the angle covers on a circle with the vertex of the angle at its center. Types of Angles There are four types of angles that one must be familiar with in order to solve for the measure of missing angles: Acute angle: An angle whose measure is less than 90 degrees. Right angle: An angle whose measure is equal to 90 degrees. Obtuse angle: An angle whose measure is greater than 90 degrees but less than 180 degrees. Straight angle: An angle whose measure is equal to 180 degrees. To find the missing angles in a given figure, one can use the following formula: Sum of all angles in a triangle = 180 degrees of all angles in a quadrilateral = 360 degrees from the given diagram, it can be seen that the three angles of the triangle add up to 180 degrees. Therefore:34 + x + 56 = 180Simplify by adding like terms:90 + x = 180Subtract 90 from both sides to isolate x:x = 90 degreesSimilarly, the four angles of the quadrilateral add up to 360 degrees. Therefore:100 + 70 + y + 120 = 360Simplify by adding like terms:290 + y = 360Subtract 290 from both sides to isolate y:y = 70 degrees

Therefore, the missing angle x is 90 degrees, and the missing angle y is 70 degrees.

To know more about Angle .

https://brainly.com/question/30693441

#SPJ8










Solve for x and y in the given expressions. Express these answers to the tenths place (i.e, one digit after the decimal point). 0.46 = log (x) 0.46 = In (y) 5.01 y 2.01 TOOLS *10

Answers

The solutions for x and y are approximately x ≈ 2.9 and y ≈ 1.6 (rounded to the tenths place).

To solve for x and y in these expressions:

0.46 = logₓ(x)

To isolate x, we can exponentiate both sides using the base 10:

10^(0.46) = x

Using a calculator, we find that x is approximately x ≈ 2.884.

0.46 = ln(y)

To isolate y, we can exponentiate both sides using the base e (Euler's number):

e^(0.46) = y

Using a calculator, we find that y is approximately y ≈ 1.586.

Therefore, the solutions are x ≈ 2.9 and y ≈ 1.6 (rounded to the tenths place).

To know more about solutions refer here:

https://brainly.com/question/12806100

#SPJ11

Other Questions
Signal Sequences: 1)A signal sequence is made of: a.Nucleotides b.Amino acids c.Lipids d.Carbohydrates 2)All protein synthesis starts on ribosomes that are located in the _________________. 3)If a molecule lacks a signal sequence it will end up in the ______________ 4)Proteins with a nuclear localization sequence (NLS) are generated on ribosomes located in the ___________________. 5)Proteins with a mitochondrial signal sequence are generated in ribosomes located in the _________________. how do you think petals protect the internal structures of a plant? Which product is greater than 63 please solve for x!!! PLSSS HELPPPP I WILLL GIVE YOU BRAINLIEST!!!!!! PLSSS HELPPPP I WILLL GIVE YOU BRAINLIEST!!!!!! PLSSS HELPPPP I WILLL GIVE YOU BRAINLIEST!!!!!! PLSSS HELPPPP I WILLL GIVE YOU BRAINLIEST!!!!!!PLSSS HELPPPP I WILLL GIVE YOU BRAINLIEST!!!!!! PLSSS HELPPPP I WILLL GIVE YOU BRAINLIEST!!!!!! PLSSS HELPPPP I WILLL GIVE YOU BRAINLIEST!!!!!! PLSS HELPP D: WILL MARK BRAINLIEST :D How can imitating an author's stylisticchoices help to understand a piece of literature and improve one's writing? Does anyone know a vb knowledge matters admin login?I'm trying to complete the Sports & Entertainment Mogul show on a number line and write: the numbers that satisfy the equation |a|=5Please Hurry!! Many cellular processes are performed by cells. A typical cell and a common process are shown in the diagram below.Which of the following is the best reason this cell is going through the process shown?to reproduce another cellto compete with another cellto maintain homeostasis of the cell to provide protection for the cell from the environment are shown in the diagram below. find a rational number between 3/5 and 4/5 ( see I just only want one so please don't give me all five)give me the process tooThanks eisenhower also targeted the federal government's continuing aid to business or what he termed___________. (a) creeping socialism (b) evil socialism (c) nationalism (d) creeping nationalist Will give brainliest Is 3. [3 marks] Use Gauss Divergence theorem to calculate S (5xi + ay 3 23 k).n dA over the sphere S: 1+ y + x2 = 9. splats J = ?? It-t? Lose - 2. [6 marks] Calculate the surface integral || G(r)da, where G = (1212 + 36)/2, the suru loob the parametrization r(u, v) = (3u, 2v, u), and 0 su 1, 0 Sv < 2. Even though Ralph and his remaining friends are exhausted and completely outnumbered, he still insists that they must do what? 7. A particle of mass 3 kg is held in equilibrium by two light unextensible strings. One string is horizontal, as shown in Figure 7.30. The tension in the horizontal string is PN and the tension in the other string is N. Find a) the value of 0 b) the value of P. which two places is aggregate generally found?floodplains and alluvial fansalluvial fans and minesmines and beneath the topsoilbeneath the topsoil and above the topsoil This next problem is going to deal with checking if someone inputs a string with a few requirements. Imagine you are prompted to input a password, but the password needs to have include uppercase and a number. In order to do this, you must look at each char in the string and Booleans to indicate certain criteria are met. Since there are 3 criteria, you should have 3 Boolean variables. The rules for the password are: Must contain at least 8 chars Must contain 1 uppercase letter Must contain 1 digit There are no restrictions with lowercase letters or special chars When considering where to use Booleans, think of it as a "flag", for each criteria. If you meet the length requirement , then the flag would change from FALSE to TRUE. Once all the flags are true, you will have a valid password. Tip: The ASCII table can be used to determine the numeric value of a particular char. You may want to create ranges of these numerical values for each criteria. Sample output +1: Enter a password: passwordi Invalid password Program Ends Sample output +2: Enter a password: P4ssw3rd Valid password Program Ends Sample output 3: Enter a password: Pasl Invalid password Let M = {a E Ra > 1). Then M is a vector space under standard addition and scalar multiplication of real numbers. False True * Let W = {a + 2x + bx Pz: a, b E R} with the standard operations in P2. Which of the following statements is true? 1+xEW W is a subspace of P2. The above is true W is not a subspace of P. Which of the following about the provision for doubtful debts is incorrect?a. None of these choices.b. It is used to provide for the small percentage of customers who may notC. The tax code used for the journal is GSTd. The provision for doubtful debts account is a contra asset. How did the climate and geography influence the ways people made money?